If m∠A = m∠B and m∠A + m∠C = m∠D, then
m∠B + m∠C = m∠D.

Answers

Answer 1

Answer:

True. The statement is also equivalent to: If m<B + m<C does not equal m<D, then m<A does not equal m<B and m<A + m<C is not equal to m<D.


Related Questions

What is the equation of a circle with its center at (−6,−3) and a radius of 12?

Answers

Answer:

(x+6)^2 + ( y +3)^2 = 144

Step-by-step explanation:

The equation of a circle is given by

(x-h)^2 +(y-k)^2 = r^2  where (h,k) is the center and r is the radius

(x- -6)^2 + ( y - -3)^2 = 12^2

(x+6)^2 + ( y +3)^2 = 144

Answer:

x² + y² + 12x +6y - 99 = 0

Step-by-step explanation:

Given :

Centre = (-6,-3) Radius = 12 ,

Using the Standard equation of circle ,

( x - h)² + (y-k)² = r² ( x +6)² + (y+3)² = 12² x² + 36 + 12x + y² + 9 + 6y = 144 x² + y² + 12x + 6y +45-144 = 0 x² + y² + 12x +6y - 99 = 0

if x=3√8, find the value of 1/x

plz its urgent

Answers

Answer:

[tex]\frac{1}{x} =\frac{1}{3\sqrt{8} } =\frac{1(\sqrt{8})}{3\sqrt{8}(\sqrt{8})} =\frac{\sqrt{8}}{3*8} =\frac{\sqrt{8}}{24} =\frac{\sqrt{(2)(2)(2)}}{24}=\frac{2\sqrt{2} }{2(12)} =\frac{\sqrt{2} }{12}[/tex]

What is 3/4 as a percentage

Answers

Answer:

75%

Step-by-step explanation:

A percent is a value out of 100.

Change the fraction [tex]\frac{3}{4}[/tex] to an equivalent fraction with the denominator equal to 100.

[tex]\frac{3}{4}[/tex] × [tex]\frac{25}{25}[/tex] = [tex]\frac{75}{100}[/tex]

This fraction represents a percent.

[tex]\frac{75}{100}[/tex] = 75%

1. The parameter "a": Compare the graphs of several different exponential growth functions in
Mathematica to discover the significance of the parameter "a". Explain in detail what the
parameter "a" tells you about the graph of an exponential growth function.

Answers

Answer:

See explanation

Step-by-step explanation:

Required

The significance of "a" in exponential function

An exponential function is represented as:

[tex]y = ab^x[/tex]

In the above equation, parameter "a" is the initial value of the function

In other words, the value of the function when x = 0

Take for instance;

[tex]y = 2*4^x[/tex]

[tex]a = 2[/tex] because when [tex]x = 0[/tex]

[tex]y = 2 * 4^0[/tex]

[tex]y = 2 * 1[/tex]

[tex]y = 2[/tex]

Another significance of parameter a is that; it is the y-intercept of the geometric function.

Mr. Mancuso plants tomatoes on 1/3 acre of land, corn on 3/4 acre, and carrots on 1/2 acre. On how many acres of land total did he plant?


1 acres


1 acres


1 acres


2 acres

Answers

Answer:

1 7/12 acres of land

Step-by-step explanation:

Tomatoes = 1/3 acre of land

Corn = 3/4 acre of land

Carrots = 1/2 acre of land

Total acres of land he planted = tomatoes + corn + carrots

= 1/3 + 3/4 + 1/2

= (4+9+6) / 12

= 19/12 acres

= 1 7/12 acres of land

Or

= 1.5833333333333 acres

Total acres of land he planted = 1 7/12 acres of land

None of the given options is correct

2. En una división el dividendo es 445, el divisor es 32, el cociente es 14 y el resto es 7. ¿Está bien hecha? Compruébalo de dos maneras diferentes

Answers

Respuesta:

El cálculo no está bien hecho.

el cociente es 13

El resto es 29

Explicación paso a paso:

Dividendo = 445

Divisor = 32

Cociente = 14

Resto = 7

445/32 = 13,90625

El cociente = 13

Resto = (13.90625 - 13) * 32

Resto = 0.90625 * 32

Resto = 29

Por lo tanto, el cálculo no se realiza correctamente ya que el cociente es 13 y el resto es 29

Lines c and d are parallel lines cut by transversal p.


Horizontal and parallel lines c and d are cut by transversal p. On line c where it intersects with line p, 4 angles are formed. Clockwise, from uppercase left, the angles are: 1, 2, 3, 4. On line d where it intersects with line p, 4 angles are formed. Clockwise, from uppercase left, the angles are: 5, 6, 7, 8.
Which must be true by the corresponding angles theorem?
A. ∠1 ≅ ∠7
B. ∠2 ≅ ∠6
C. ∠3 ≅ ∠5
D. ∠5 ≅ ∠7

Answers

Answer:

Option (B).

Step-by-step explanation:

Here there are two parallel lines c and d cuts by a transversal p.

The angles are formed as shown in diagram.

Here,

[tex]\angle 1 = \angle 7 (alternate)\\\\\angle 2 = \angle 6 (corresponding)\\\\\angle 3 = \angle 5 (alternate)\\\\\angle 5 = \angle 7 (alternate)[/tex]

So, the option (B) is correct.

How many tacos could you buy on a Wednesday with 20 dollars and tacos cost 50. cents

Answers

Answer:

40 tacos

Step-by-step explanation:

Step 1:  Determine how many tacos you can buy

Cost of one taco:  $0.50

Money in wallet:  $20

So for every $1 you can buy 2 tacos.  Therefore, 20 * 2 = 40 tacos

Answer:  40 tacos

Answer:40 tacos

Step-by-step explanation: because yeah

In a shipment of airplane parts, 6% are known to be defective. If 42 parts are found to be defective, how many parts are in the shipment?

Answers

Answer:

700 parts

Step-by-step explanation:

To find the total amount of parts in the shipment, all we need to do is divide.

6% = 0.06

42 / 0.06 = 700

Best of Luck!

Someone help me pls ..

Answers

Answer:

because they are both in the circle

Step-by-step explanation:

How much money invested at 3% compounded monthly for 3 years will yield $520?
$179.42
$475.30
$358.84
$148.78

Answers

Answer:

Step-by-step explanation:

Use this formula:

[tex]A(t)=P(1+\frac{r}{n})^{nt}[/tex] where A(t) is the amount after the compounding is done, P is the initial investment (our unknown), r is the interest rate in decimal form, n is the number of compoundings per year, and t is the time in years. Filling in:

[tex]520=P(1+\frac{.03}{12})^{(12)(3)}[/tex] and simplifying that a bit:

[tex]520=P(1+.0025)^{36[/tex] and a bit more:

[tex]520=P(1.0025)^{36[/tex] and even  bit more:

520 = P(1.094551401) and divide to get

P = $475.30

I’m stuck on this one help anyone?

Answers

Answer:

just add a small amount to the 2.8 and square the result

Step-by-step explanation:

x x^2

2.8 7.84

2.81 7.8961

2.82 7.9524

2.83 8.0089

2.84 8.0656

2.85 8.1225

2.86 8.1796

2.87 8.2369

Let j=+5 - 5+ |-5 x 1/5
What is the value of+J?

Answers

Answer:

j=|x|

Step-by-step explanation:

Solve EFD. Round the answers to the nearest hundredth.
A. m F ≈ 26, m D ≈ 64.01, FD = 7,921
B. m F ≈ 26, m D ≈ 64.01, FD = 89
C. m F ≈ 64.01, m D ≈ 26, FD = 89
D. m F ≈ 64.01, m D ≈ 26, FD = 7,921

Answers

Answer:

Option B

<F = 26°

<D = 64.01°

FD = 89

Answered by GAUTHMATH

For right triangle EFD,  m ∠F ≈ 26°, m ∠D ≈ 64.01° and FD = 89

The correct answer is an option (B)

What is hypotenuse?

It is the longest side of the right triangle.

What is Pythagoras theorem?

For a right triangle,

[tex]a^{2}+ b^{2} = c^{2}[/tex], where c is hypotenuse and a, b area other two sides of the right triangle

For given example,

We have been given a right triangle EFD with hypotenuse FD.

Also, EF = 80, ED = 39

Using the Pythagoras theorem,

[tex]\Rightarrow FD^{2}= EF^{2} + ED^{2}\\\\ \Rightarrow FD^{2}= 80^{2} + 39^{2}\\\\ \Rightarrow FD^2 = 6400 + 1521\\\\ \Rightarrow FD^2 = 7921\\\\\Rightarrow FD = 89[/tex]

Consider, sin(F)

[tex]\Rightarrow sin(F)=\frac{ED}{FD} \\\\\Rightarrow sin(F)=\frac{39}{89}\\\\ \Rightarrow sin(F)=0.4382\\\\\Rightarrow \angle F=sin^{-1}(0.4382)\\\\\Rightarrow \angle F=25.98^{\circ}\\\\\Rightarrow \angle F\approx 26^{\circ}[/tex]

Now, consider sin(D)

[tex]\Rightarrow sin(D)=\frac{FE}{FD}\\\\ \Rightarrow sin(D)=\frac{80}{89}\\\\ \Rightarrow \angle D = sin^{-1}(0.8988)\\\\\Rightarrow \angle D = 64.009^{\circ}\\\\\Rightarrow \angle D \approx 64.01^{\circ}[/tex]

Therefore, for right triangle EFD,  m ∠F ≈ 26°, m ∠D ≈ 64.01° and FD = 89

The correct answer is an option (B)

Learn more about Pythagoras theorem here:

https://brainly.com/question/343682

#SPJ2

Si un proyectil asciende verticalmente, y después de 3 segundos alcanza su altura máxima, calcule la velocidad que lleva a la mitad de su trayectoria descendente

Answers

Answer:

The speed is 20.8  m/s

Step-by-step explanation:

If a projectile ascends vertically, and after 3 seconds it reaches its maximum height, calculate the velocity that it carries to the middle of its downward trajectory

Let the maximum height is h and initial velocity is u.

From first equation of motion

v = u + at

0 = u - g x 3

u = 3 g.....(1)

Use third equation of motion

[tex]v^2 = u^2 - 2 gh \\\\0 = 9 g^2 - 2 gh \\\\h = 4.5 g[/tex]

Let the speed at half the height is v'.

[tex]v^2 = u^2 + 2 gh \\\\v'^2 = 0 + 2 g\times 2.25 g\\\\v'^2 = 4.5\times 9.8\times9.8\\\\v' = 20.8 m/s[/tex]


Choose the correct description of the graph of the inequality X - 3 greater than or equal to 25

A. Open circle on 8, shading to the left
B. Closed circle on 8, shading to the left.
C. Open circle on 8, shading to the right.
D. Closed circle on 8, shading to the right.

I’m pretty sure it’s D

Answers

Answer:

D. Closed circle on 8, shading to the right.

Help Please Now!!!
Find the volume Of The Rectangle Prism

Answers

Answer:

180m cubed

Step-by-step explanation:

Multiply the length, width, and height together: 6×6×5=180

Answer:

Formula of a rectangular prism:

L x W x H

Now we place in the numbers and solve:

5 x 6 x 6 = 180 m3

What is the common difference of the arithmetic sequence-20,-16,-12,-18

Answers

Answer:

common difference = 4

Step-by-step explanation:

common difference is the difference between the successive term and its preceding term.

let's take the successive term of -20 that is - 16

common difference (d) = successive term - preceeding term

= -16 -(-20)

= -16 + 20

= 4

if we take the successive term of -16 that is -12

we'll get the same common difference.

d = -12 -(-16)

d = -12 + 16

d = 4

this means that the common difference for an AP remains constant.

help me out (geometry)

Answers

Answer:

Step-by-step explanation:

d and b meet at a 90 degree angle ( as shown by the box)

The lines are perpendicular (⊥)

plsssssssssss helppppppppp i want it right now pls​

Answers

Answer:

hope this helps you

have a great day

Jessica ate 7 /10 of her orange before lunch and 1/10 of her orange after lunch. How much of her orange did she eat?

Answers

Answer:

8/10

Step-by-step explanation:

Find the discernment and the numbers of the number of real roots for this equation.
x^2+3x+8=0

Answers

Answer:  2 distinct complex solutions (ie non real solutions).

Work Shown:

The given equation is in the form ax^2+bx+c = 0, so

a = 1, b = 3, c = 8

Plug those into the formula below to find the discriminant

D = b^2 - 4ac

D = 3^2 - 4(1)(8)

D = -23

The discriminant is negative, so we get two nonreal solutions. The two solutions are complex numbers in the form a+bi, where a & b are real numbers and [tex]i = \sqrt{-1}[/tex]. The two solutions are different from one another.

Answer:

Discriminant: -23

Number of real roots: 0

Step-by-step explanation:

For a quadratic in standard form [tex]ax^2+bx+c[/tex], the discriminant is given by [tex]b^2-4ac[/tex].

In [tex]x^2+3x+8[/tex], assign:

[tex]a\implies 1[/tex] [tex]b\implies 3[/tex] [tex]c\implies 8[/tex]  

The discriminant is therefore:

[tex]3^2-4(1)(8)=9-32=\boxed{-23}[/tex]

For any quadratic:

If the discriminant is greater than 0, the quadratic has two real rootsIf the discriminant is equal to 0, the quadratic has one real rootIf the discriminant is less than 0, the quadratic as no real roots

Since the quadratic in the question has a discriminant less than 0, there are no real solutions to this quadratic.

Please help me Find PA.

Answers

PO is 5 by using Pythagorean theorems and it forming a 3,4,5 triangle so PA is PO + OA so it is 8

Find the area to the left of z = 0.25.
A. 0.6012 B. 0.5987 C. 0.4013 D.0.3988

Answers

Answer:

.5987

Step-by-step explanation:

Use a ztable and find .25 (pic below)

which of the following are exterior angles?

Answers

Answer:

A, B, E

Step-by-step explanation:

Exterior angles are angles that are outside the shape. In this case, angle 4, 3 and 2 are exterior angles.

The square root of 0.25 is 0.5 which is a greater number. Give another number whose square root is larger than the number and explain why.

Answers

Answer:

Another example of a number who's square root is greater than the number is  [tex]\sqrt{0.49}[/tex] which is 0.7

Step-by-step explanation:

This square root is larger than the number because it is a decimal. When you multiply a decimal by a decimal, the decimal point becomes greater. For example: 0.7 multiplied by 0.7 equals 0.49 which has 2 decimal places, while 0.7 only has one.

Find the value of x.

A. 99
B. 9
C. 90
D. 11
ILL GIVE BRAINLIEST

Answers

Answer:

B) 9

Step-by-step explanation:

Because there's a square between the 2 angles, that means these angles are complementary (angles that add up to 90°). So:

5x - 9 + 6x = 90

11x - 9 = 90

11x = 90 + 9

11x = 99

x = 9

Answer:

B.9

Step-by-step explanation:

The way to solve this is by noticing that these angles are complementary(they add up to 90 degrees). So you add the equations together and equal them to 90. 5x-9+6x=90.Then you solve to find that x=9.

Find the line’s slope and a point on the line
Y-4=-3/4(x+5)

Answers

Answer:

The slope is -3/4 and a point on the line is (-5,4)

Step-by-step explanation:

This equation is in point slope form

y -y1 = m(x-x1)

where m is the slope and (x1,y1) is a point on the line

Y-4=-3/4(x+5)

Y-4=-3/4(x -  -5)

The slope is -3/4 and a point on the line is (-5,4)

40 points Please help!!!
What is the volume of this regular prism?
48.55 cubic inches
55.8 cubic inches
9.7 cubic inches
24.28 cubic inches

Answers

Answer:

V = 24.28 in ^3

Step-by-step explanation:

The area of the base is

A =5/2 × s × a  where s is the side length and a is the apothem

A = 5/2 ( 2.13) * .87

A = 4.63275

The volume is

V = Bh  where B is the area of the base and h is the height

V = 4.63275 ( 5.24)

V =24.27561 in^3

Rounding to the  hundredth

V = 24.28 in ^3

What is “8 - 4(-x + 5)” equivalent too?

Answers

Answer:

4x -12

Step-by-step explanation:

8 - 4(-x + 5)

Distribute

8 -4(-x) -4(5)

8 +4x -20

4x -12

answer 4( - 3 + x)

factor expression

4(2 - ( - x + 5)4(2 + x - 5)

answer

[tex]4( - 3 + x)[/tex]

simplify the expression

[tex]8 - 4( - x + 5)[/tex]

answer

[tex] - 12 + 4x[/tex]

Other Questions
A teacher calculates for the test grades inClass A, mean = 32 and sd = 4Class B, mean = 32 and sd = 8a. If the teacher was going to guess what any student in his/her class would earn, what is the best scoreto guess?b. Which of the classes has more consistency in their scores? Why? Given JIH~ LMN. what is the value of LN? The summer camps had a field trip from the campus to Fragrance Hill. They traveled at an average speed of 65 km/h in the first 2 hours. After that, traveled at another average speed of 78 km/h. If the distance between the campus and Fragrance Hill is 364 km, what was the total time for the field trip? what is the impact germany had in their country? Help me pls Someone plz help me and show the work 10 points! Write the y-intercept of the line John and Sarah both apply for the same promotion in their news organization. John gets the promotion and Sarah decides that the new job wasn't really that important to her. Sarah is exhibiting which of Freudian defense mechanism?a. reaction formation.b. projection.c. rationalization.d. denial. HELP! (25 points) i think i might know the answer If an airplane flies North over the North Pole to get from New York City to Stockholm, Sweden, its course saved fuel and time based on the principle of you decide to work part time at a local supermarket. The job pays eight dollars and 60 per hour and you work 20 hours per week. Your employer withhold 10% of your gross pay federal taxes, 7.65% for FICA taxes, and 5% for state taxes Which system is equivalent to y=-2x^2 y=x-2 a) y=-2y^2 x=y-2b) y=-2y^2+2 x=y+2c) y=-2y^2-8y-8 x=y+2d) y=-2y^2+8y-8 x=y-2 andSelect all that apply.12 of 25The order converts the leader's plan into action. The five paragraph format helps to promote Grant is a member of a book club. He pays a $10 yearly membership fee and can purchase books through the club for $2.75 each. His total annual cost is a function of the number of books that he purchases in a year. Let b represent the number of books he purchases in a year. Which function, C(b), represents his yearly cost explain how conduction occur Are the phrases very interestingandconsuming synonymous An industrial psychologist consulting with a chain of music stores knows that the average number of complaints management receives each month throughout the industry is 4, but the variance is unknown. Nine of the chain's stores were randomly selected to record complaints for one month; they received 2, 4, 3, 5, 0, 2, 5, 1, and 5 complaints. Using the .05 significance level, is the number of complaints received by the chain different from the number of complaints received by music stores in general?1. Use the five steps of hypothesis testing.2. Sketch the distributions involved3. Explain the logic of what you did to a person who is familiar with hypothesis testing, but knows nothing about t tests of any kind. Be sure to explain how this problem differs from a problem with a known population variance and a single sample. In preparing its bank reconciliation for the month of April 2018, Crane, Inc. has available the following information. Balance per bank statement, 4/30/18 $78200 NSF check returned with 4/30/18 bank statement 950 Deposits in transit, 4/30/18 9600 Outstanding checks, 4/30/18 10300 Bank service charges for April 50 What should be the adjusted cash balance at April 30, 2018 Plz help!! I need it ASAP Write a paragraph about the happiest day in your life.